Stay ahead of learning milestones! Enroll in a class over the summer!

G
Topic
First Poster
Last Poster
Constant Angle Sum
i3435   6
N 25 minutes ago by bin_sherlo
Source: AMASCIWLOFRIAA1PD (mock oly geo contest) P3
Let $ABC$ be a triangle with circumcircle $\Omega$, $A$-angle bisector $l_A$, and $A$-median $m_A$. Suppose that $l_A$ meets $\overline{BC}$ at $D$ and meets $\Omega$ again at $M$. A line $l$ parallel to $\overline{BC}$ meets $l_A$, $m_A$ at $G$, $N$ respectively, so that $G$ is between $A$ and $D$. The circle with diameter $\overline{AG}$ meets $\Omega$ again at $J$.

As $l$ varies, show that $\angle AMN + \angle DJG$ is constant.

MP8148
6 replies
i3435
May 11, 2021
bin_sherlo
25 minutes ago
NEPAL TST 2025 DAY 2
Tony_stark0094   8
N 40 minutes ago by cursed_tangent1434
Consider an acute triangle $\Delta ABC$. Let $D$ and $E$ be the feet of the altitudes from $A$ to $BC$ and from $B$ to $AC$ respectively.

Define $D_1$ and $D_2$ as the reflections of $D$ across lines $AB$ and $AC$, respectively. Let $\Gamma$ be the circumcircle of $\Delta AD_1D_2$. Denote by $P$ the second intersection of line $D_1B$ with $\Gamma$, and by $Q$ the intersection of ray $EB$ with $\Gamma$.

If $O$ is the circumcenter of $\Delta ABC$, prove that $O$, $D$, and $Q$ are collinear if and only if quadrilateral $BCQP$ can be inscribed within a circle.

$\textbf{Proposed by Kritesh Dhakal, Nepal.}$
8 replies
Tony_stark0094
Apr 12, 2025
cursed_tangent1434
40 minutes ago
Interesting inequalities
sqing   4
N an hour ago by sqing
Source: Own
Let $ a,b,c\geq  0 $ and $  ab+bc+ca+abc=4$ . Prove that
$$k(a+b+c) -ab-bc\geq 4\sqrt{k(k+1)}-(k+4)$$Where $ k\geq \frac{16}{9}. $
$$ \frac{16}{9}(a+b+c) -ab-bc\geq  \frac{28}{9}$$
4 replies
1 viewing
sqing
3 hours ago
sqing
an hour ago
NEPAL TST DAY 2 PROBLEM 2
Tony_stark0094   6
N an hour ago by cursed_tangent1434
Kritesh manages traffic on a $45 \times 45$ grid consisting of 2025 unit squares. Within each unit square is a car, facing either up, down, left, or right. If the square in front of a car in the direction it is facing is empty, it can choose to move forward. Each car wishes to exit the $45 \times 45$ grid.

Kritesh realizes that it may not always be possible for all the cars to leave the grid. Therefore, before the process begins, he will remove $k$ cars from the $45 \times 45$ grid in such a way that it becomes possible for all the remaining cars to eventually exit the grid.

What is the minimum value of $k$ that guarantees that Kritesh's job is possible?

$\textbf{Proposed by Shining Sun. USA}$
6 replies
Tony_stark0094
Apr 12, 2025
cursed_tangent1434
an hour ago
NEPAL TST DAY-2 PROBLEM 1
Tony_stark0094   9
N an hour ago by cursed_tangent1434
Let the sequence $\{a_n\}_{n \geq 1}$ be defined by
\[
a_1 = 1, \quad a_{n+1} = a_n + \frac{1}{\sqrt[2024]{a_n}} \quad \text{for } n \geq 1, \, n \in \mathbb{N}
\]Prove that
\[
a_n^{2025} >n^{2024}
\]for all positive integers $n \geq 2$.

$\textbf{Proposed by Prajit Adhikari, Nepal.}$
9 replies
Tony_stark0094
Apr 12, 2025
cursed_tangent1434
an hour ago
Inspired by Omerking
sqing   1
N an hour ago by lbh_qys
Source: Own
Let $ a,b,c>0 $ and $  ab+bc+ca\geq \dfrac{1}{3}. $ Prove that
$$  ka+ b+kc\geq \sqrt{\frac{4k-1}{3}}$$Where $ k\geq 1.$$$  4a+ b+4c\geq \sqrt{5}$$
1 reply
sqing
2 hours ago
lbh_qys
an hour ago
Weird Inequality Problem
Omerking   4
N 2 hours ago by sqing
Following inequality is given:
$$3\geq ab+bc+ca\geq \dfrac{1}{3}$$Find the range of values that can be taken by :
$1)a+b+c$
$2)abc$

Where $a,b,c$ are positive reals.
4 replies
Omerking
Yesterday at 8:56 AM
sqing
2 hours ago
A Projection Theorem
buratinogigle   2
N 2 hours ago by wh0nix
Source: VN Math Olympiad For High School Students P1 - 2025
In triangle $ABC$, prove that
\[ a = b\cos C + c\cos B. \]
2 replies
buratinogigle
6 hours ago
wh0nix
2 hours ago
Turbo's en route to visit each cell of the board
Lukaluce   18
N 3 hours ago by yyhloveu1314
Source: EGMO 2025 P5
Let $n > 1$ be an integer. In a configuration of an $n \times n$ board, each of the $n^2$ cells contains an arrow, either pointing up, down, left, or right. Given a starting configuration, Turbo the snail starts in one of the cells of the board and travels from cell to cell. In each move, Turbo moves one square unit in the direction indicated by the arrow in her cell (possibly leaving the board). After each move, the arrows in all of the cells rotate $90^{\circ}$ counterclockwise. We call a cell good if, starting from that cell, Turbo visits each cell of the board exactly once, without leaving the board, and returns to her initial cell at the end. Determine, in terms of $n$, the maximum number of good cells over all possible starting configurations.

Proposed by Melek Güngör, Turkey
18 replies
Lukaluce
Monday at 11:01 AM
yyhloveu1314
3 hours ago
Perhaps a classic with parameter
mihaig   1
N 3 hours ago by LLriyue
Find the largest positive constant $r$ such that
$$a^2+b^2+c^2+d^2+2\left(abcd\right)^r\geq6$$for all reals $a\geq1\geq b\geq c\geq d\geq0$ satisfying $a+b+c+d=4.$
1 reply
mihaig
Jan 7, 2025
LLriyue
3 hours ago
Connected graph with k edges
orl   26
N 4 hours ago by Maximilian113
Source: IMO 1991, Day 2, Problem 4, IMO ShortList 1991, Problem 10 (USA 5)
Suppose $ \,G\,$ is a connected graph with $ \,k\,$ edges. Prove that it is possible to label the edges $ 1,2,\ldots ,k\,$ in such a way that at each vertex which belongs to two or more edges, the greatest common divisor of the integers labeling those edges is equal to 1.

Note: Graph-Definition. A graph consists of a set of points, called vertices, together with a set of edges joining certain pairs of distinct vertices. Each pair of vertices $ \,u,v\,$ belongs to at most one edge. The graph $ G$ is connected if for each pair of distinct vertices $ \,x,y\,$ there is some sequence of vertices $ \,x = v_{0},v_{1},v_{2},\cdots ,v_{m} = y\,$ such that each pair $ \,v_{i},v_{i + 1}\;(0\leq i < m)\,$ is joined by an edge of $ \,G$.
26 replies
orl
Nov 11, 2005
Maximilian113
4 hours ago
Nepal TST 2025 DAY 1 Problem 1
Bata325   7
N Yesterday at 6:50 AM by cursed_tangent1434
Source: Nepal TST 2025 p1
Consider a triangle $\triangle ABC$ and some point $X$ on $BC$. The perpendicular from $X$ to $AB$ intersects the circumcircle of $\triangle AXC$ at $P$ and the perpendicular from $X$ to $AC$ intersects the circumcircle of $\triangle AXB$ at $Q$. Show that the line $PQ$ does not depend on the choice of $X$.

(Shining Sun, USA)
7 replies
Bata325
Apr 11, 2025
cursed_tangent1434
Yesterday at 6:50 AM
Nepal TST 2025 DAY 1 Problem 1
G H J
G H BBookmark kLocked kLocked NReply
Source: Nepal TST 2025 p1
The post below has been deleted. Click to close.
This post has been deleted. Click here to see post.
Bata325
8 posts
#1 • 3 Y
Y by khan.academy, cubres, Mathdreams
Consider a triangle $\triangle ABC$ and some point $X$ on $BC$. The perpendicular from $X$ to $AB$ intersects the circumcircle of $\triangle AXC$ at $P$ and the perpendicular from $X$ to $AC$ intersects the circumcircle of $\triangle AXB$ at $Q$. Show that the line $PQ$ does not depend on the choice of $X$.

(Shining Sun, USA)
This post has been edited 6 times. Last edited by Bata325, Apr 13, 2025, 2:54 AM
Reason: italics
Z K Y
The post below has been deleted. Click to close.
This post has been deleted. Click here to see post.
Mathdreams
1464 posts
#2 • 1 Y
Y by cubres
My problem! :)

Shocking that my olympiad problem writing debut is a geometry problem. People that know me will know how bad I am at geo. :wacko:

Solution
This post has been edited 2 times. Last edited by Mathdreams, Apr 12, 2025, 8:33 PM
Z K Y
The post below has been deleted. Click to close.
This post has been deleted. Click here to see post.
wassupevery1
317 posts
#3 • 1 Y
Y by cubres
Solution
Z K Y
The post below has been deleted. Click to close.
This post has been deleted. Click here to see post.
Tony_stark0094
61 posts
#4 • 1 Y
Y by cubres
The points used in this proof are as shown in the figure $O_1,O_2,O_3$ are the centres of the circles $\odot ABC, \odot ACX, \odot ABX$ respectively
I claim that the line $PQ$ always passes through the circumcentre of $\Delta ABC$
proof:
Define phantom points $P'$ and $Q'$ such that $P'=\odot ACX \cap AO_1$ and $Q'= \odot ABX \cap AO_1$
now we know that $O_1O_2$ and $O_1O_3$ are the perpendicular bisectors of the radial axes $AC$ and $AB$
$\angle AP'X= \angle ACX = \angle C$ and $\angle AO_1O_3= \angle AO_1E= \angle C$ hence $O_1O_3 \parallel P'X \implies P'X \perp AB$
and
$\angle AQ'D=\angle ABX =\angle B$ and $\angle AO_1E= \angle B$ so $O_1O_2 \parallel  Q'D \implies Q'D \perp AC$
hence $P' \equiv P $ and $Q' \equiv Q$ and the required claim is proved
Attachments:
Z K Y
The post below has been deleted. Click to close.
This post has been deleted. Click here to see post.
Tony_stark0094
61 posts
#5 • 2 Y
Y by Mathdreams, cubres
Mathdreams wrote:
My problem! :)

Shocking that my olympiad problem writing debut is a geometry problem. People that know me will know how bad I am at geo. :wacko:

Solution

I think there should be 180 - 2ACB in your third line
This post has been edited 1 time. Last edited by Tony_stark0094, Apr 13, 2025, 2:41 AM
Reason: pllllll
Z K Y
The post below has been deleted. Click to close.
This post has been deleted. Click here to see post.
ThatApollo777
71 posts
#6 • 1 Y
Y by cubres
Claim : $AO$ is the required line where $O$ is circumcenter of $ABC$

Pf. $\measuredangle BAP = 90 - \measuredangle APX = 90 - \measuredangle ACX = 90 - \measuredangle ACB = \measuredangle BAO$ hence $P$ lies on $AO$ and similarly $Q$ must also lie on AO so we are done.
This post has been edited 3 times. Last edited by ThatApollo777, Apr 12, 2025, 3:34 PM
Reason: clarity
Z K Y
The post below has been deleted. Click to close.
This post has been deleted. Click here to see post.
Mathdreams
1464 posts
#7 • 1 Y
Y by cubres
@2above Fixed!
Z K Y
The post below has been deleted. Click to close.
This post has been deleted. Click here to see post.
cursed_tangent1434
586 posts
#8 • 1 Y
Y by GeoKing
The idea is that both points $P$ and $Q$ lie on the line $\overline{AO}$ where $O$ is the circumcenter of $\triangle ABC$. To see why,
\[\measuredangle BAQ = \measuredangle CXQ = 90 + \measuredangle BCA\]which implies that $Q$ lies on $\overline{AO}$. Similarly,
\[\measuredangle CAQ = \measuredangle CXP = 90 + \measuredangle CBA\]which implies that $P$ lies on $\overline{AO}$ and thus the line $PQ$ does not depend on the choice of $X$.
Z K Y
N Quick Reply
G
H
=
a